LSAT and Law School Admissions Forum

Get expert LSAT preparation and law school admissions advice from PowerScore Test Preparation.

User avatar
 Dave Killoran
PowerScore Staff
  • PowerScore Staff
  • Posts: 5852
  • Joined: Mar 25, 2011
|
#41344
Complete Question Explanation
(The complete setup for this game can be found here: lsat/viewtopic.php?t=10632)

The correct answer choice is (D)

This type of question would usually take a great deal of time because of the “ifs” present in each answer choice. Thus, generally it would be wise to do this type of question at the end of the game, after you have gathered as many hypotheticals as possible. In this case the hypothetical from question #2 eliminates answer choice (E). No other hypothetical from the game eliminates an answer choice, and you are forced to examine each answer choice on its own merits. Answer choice (D) is correct since U on Thursday fulfills the two-on-one-day rule, forcing the KO block into Monday, Tuesday, or Wednesday. This leaves the H :longline: Z sequence, and since no more doubles are allowed, H must go on Monday or Wednesday, leaving only Z to go on Friday.
 S_Hernandez52
  • Posts: 13
  • Joined: Feb 28, 2017
|
#33572
I was wondering if there is a quicker way to get to question #3. I wrote out the scenarios in order to get the right answers. I know this wasn't the most efficient way to get to the answer. Was wondering if there are any tips or advice for attacking must be true answers inside this logic game. Thank you!
 Robert Carroll
PowerScore Staff
  • PowerScore Staff
  • Posts: 1787
  • Joined: Dec 06, 2013
|
#33573
S,

This "Five If" question type can be very time consuming. Because you can choose to try out the answer choices in any order you want, a useful strategy may be to look first at answer choices that involve a lot of restriction. In this game, things that would create restriction might be as follows:

-anything that determines completely which day has 2 students

-anything involving K or O, as the presence of one of those on a particular day severely restricts where the other one can go

-anything involving H and Z, especially if H is late in the week or Z is early in the week, as this severely restricts where the other can go.

-finally, notice how every sufficient condition puts U on a different day. What's the most restrictive place for U to go?

Because J is already on Thursday, anyone else being on that day completely fixes the numbers of students on each day. Answer choice (D) puts U there as well. This leaves K and O to have to go M-W in order to be next to each other. H must be before Z, so H must be one of these three days, forcing Z to be on Friday, as required.

Robert Carroll
User avatar
 parisielvirac
  • Posts: 30
  • Joined: Jan 20, 2021
|
#86783
why is answer choice C incorrect?
it would make I and U go together on Thursday reserving the two to one day training.
leaving K,O,H to go Monday, Tuesday and Wednesday (in that order), and forcing/leaving Z to go Friday.
why was this answer choice eliminated?
User avatar
 parisielvirac
  • Posts: 30
  • Joined: Jan 20, 2021
|
#86785
parisielvirac wrote: Mon May 03, 2021 10:49 am why is answer choice C incorrect?
it would make I and U go together on Thursday reserving the two to one day training.
leaving K,O,H to go Monday, Tuesday and Wednesday (in that order), and forcing/leaving Z to go Friday.
why was this answer choice eliminated?
never mind I read the wrong answer, got it
 Anfernee9320
  • Posts: 8
  • Joined: Jan 15, 2020
|
#89161
Why is answer choice C incorrect? Is the idea to form a sequence that complies with the global and local rules?
 Robert Carroll
PowerScore Staff
  • PowerScore Staff
  • Posts: 1787
  • Joined: Dec 06, 2013
|
#89169
Anfernee,

I think I can make all the rules true and make the sufficient condition of answer choice (C) true, but not the necessary condition, showing that answer choice (C) isn't a true conditional - its sufficient condition can be true without its necessary condition. This would show it does not have to be true.

I think this hypothetical works:

Monday: O
Tuesday: K
Wednesday: U
Thursday: HI
Friday: Z

Robert Carroll
 Anfernee9320
  • Posts: 8
  • Joined: Jan 15, 2020
|
#89170
So the idea for questions like this one is to identify counterattacks where the sufficient condition is met, but the necessary condition is not met? Or vice versa? Or both are applicable?

In other words, if I am able to prove that U's session can be scheduled on Wednesday without Z's session being scheduled on Tuesday or vice versa, does that warrant crossing out answer choice C?
 Robert Carroll
PowerScore Staff
  • PowerScore Staff
  • Posts: 1787
  • Joined: Dec 06, 2013
|
#89171
Anfernee,

It's important that you're testing that the sufficient can be true without the necessary. If that's possible, the answer is wrong. If the necessary can be true without the sufficient, that doesn't prove anything, so don't spend time doing that! A necessary condition of a conditional can be true without the sufficient.

Robert Carroll
User avatar
 ridolph.lauren
  • Posts: 16
  • Joined: Feb 20, 2022
|
#94793
Robert,

So whenever we encounter these "if" time consuming choices in a question of this nature that requires a "must be true" answer, is it always safe to approach it by trying to quickly figure out which answer shows that the sufficient can't be without the necessary condition as stated in that particular answer choice? And with that, if we spot it immediately, we can safely move on so as to not waste anymore time on that question?

Thank you,

Lauren

Get the most out of your LSAT Prep Plus subscription.

Analyze and track your performance with our Testing and Analytics Package.